2021 AMC 10B Problems/Problem 1

Revision as of 17:07, 11 February 2021 by Pureswag (talk | contribs)

How many integer values of $x$ satisfy $|x|<3\pi$?

$\textbf{(A)} ~9 \qquad\textbf{(B)} ~10 \qquad\textbf{(C)} ~18 \qquad\textbf{(D)} ~19 \qquad\textbf{(E)} ~20$